Diễn Đàn MathScopeDiễn Đàn MathScope
  Diễn Đàn MathScope
Ghi Danh Hỏi/Ðáp Community Lịch

Go Back   Diễn Đàn MathScope > Sơ Cấp > Đại Số và Lượng Giác > Các Bài Toán Đã Được Giải

News & Announcements

Ngoài một số quy định đã được nêu trong phần Quy định của Ghi Danh , mọi người tranh thủ bỏ ra 5 phút để đọc thêm một số Quy định sau để khỏi bị treo nick ở MathScope nhé !

* Nội quy MathScope.Org

* Một số quy định chung !

* Quy định về việc viết bài trong diễn đàn MathScope

* Nếu bạn muốn gia nhập đội ngũ BQT thì vui lòng tham gia tại đây

* Những câu hỏi thường gặp

* Về việc viết bài trong Box Đại học và Sau đại học


Ðề tài đã khoá Gởi Ðề Tài Mới
 
Ðiều Chỉnh Xếp Bài
Old 31-08-2010, 08:33 PM   #166
crystal_liu
+Thành Viên+
 
crystal_liu's Avatar
 
Tham gia ngày: Aug 2010
Đến từ: Akaban
Bài gởi: 353
Thanks: 94
Thanked 199 Times in 141 Posts
------------------------------
[/QUOTE],có thể trình bày chi tiết không ,lời giải p,q,r ấy .mình kém khoản này lắm
[RIGHT][I][B]Nguồn: MathScope.ORG[/B][/I][/RIGHT]
 
__________________
BEAST
crystal_liu is offline  
The Following User Says Thank You to crystal_liu For This Useful Post:
IMO 2010 (27-11-2010)
Old 31-08-2010, 08:40 PM   #167
shido_soichua
Maths is my life
 
shido_soichua's Avatar
 
Tham gia ngày: Oct 2009
Đến từ: Ninh Bình
Bài gởi: 300
Thanks: 31
Thanked 132 Times in 76 Posts
Gửi tin nhắn qua Yahoo chát tới shido_soichua
Mời mọi người thử sức với bài này:
Cho $a,b,c\in \left [ 0,1 \right ] $. Cm bất đẳng thức sau đây:
$3+a^{2}b+b^{2}c+c^{2}a\geq 2(a^{3}+b^{3}+^{3}) $
[RIGHT][I][B]Nguồn: MathScope.ORG[/B][/I][/RIGHT]
 
__________________
http://luongvantuy.org/forum.php
Chuyên Văn - Lương Văn Tụy
shido_soichua is offline  
The Following User Says Thank You to shido_soichua For This Useful Post:
IMO 2010 (27-11-2010)
Old 31-08-2010, 08:44 PM   #168
crystal_liu
+Thành Viên+
 
crystal_liu's Avatar
 
Tham gia ngày: Aug 2010
Đến từ: Akaban
Bài gởi: 353
Thanks: 94
Thanked 199 Times in 141 Posts
Ta phải CM:$(x^3+2y^3+2)(y^3+2z^3+2)(z^3+2x^3+2) \le 125 $ (Đ)
------------------------------
Chỉ rõ chỗ này xem nào ,hình như có vấn đề
[RIGHT][I][B]Nguồn: MathScope.ORG[/B][/I][/RIGHT]
 
__________________
BEAST
crystal_liu is offline  
The Following User Says Thank You to crystal_liu For This Useful Post:
IMO 2010 (27-11-2010)
Old 31-08-2010, 08:46 PM   #169
353535
Banned
 
Tham gia ngày: Jul 2010
Đến từ: LVT_NB
Bài gởi: 134
Thanks: 3
Thanked 61 Times in 38 Posts
Gửi tin nhắn qua Yahoo chát tới 353535
Trích:
Nguyên văn bởi shido_soichua View Post
Mời mọi người thử sức với bài này:
Cho $a,b,c\in \left [ 0,1 \right ] $. Cm bất đẳng thức sau đây:
$3+a^{2}b+b^{2}c+c^{2}a\geq 2(a^{3}+b^{3}+^{3}) $
ta có:
------------------------------
Trích:
Nguyên văn bởi crystal_liu View Post
Chỉ rõ chỗ này xem nào ,hình như có vấn đề

[RIGHT][I][B]Nguồn: MathScope.ORG[/B][/I][/RIGHT]
 

thay đổi nội dung bởi: 353535, 31-08-2010 lúc 08:54 PM Lý do: Tự động gộp bài
353535 is offline  
The Following User Says Thank You to 353535 For This Useful Post:
IMO 2010 (27-11-2010)
Old 31-08-2010, 09:06 PM   #170
crystal_liu
+Thành Viên+
 
crystal_liu's Avatar
 
Tham gia ngày: Aug 2010
Đến từ: Akaban
Bài gởi: 353
Thanks: 94
Thanked 199 Times in 141 Posts
[QUOTE=353535;63855]ta có:[HINT]$a(a-1)(b-1) =a^2b+1-a^2-b \ge 0 $
Chỗ này phải là $(a^2-1)(b-1)=a^2b-a^2-b+1 $giải tương tự như trên thì chỉ thấy dấu =khi a=b=c=1 mà thực tế còn a=b=1,c=0 và các hoán vị
[RIGHT][I][B]Nguồn: MathScope.ORG[/B][/I][/RIGHT]
 
__________________
BEAST

thay đổi nội dung bởi: crystal_liu, 31-08-2010 lúc 09:51 PM
crystal_liu is offline  
The Following User Says Thank You to crystal_liu For This Useful Post:
IMO 2010 (27-11-2010)
Old 31-08-2010, 09:10 PM   #171
353535
Banned
 
Tham gia ngày: Jul 2010
Đến từ: LVT_NB
Bài gởi: 134
Thanks: 3
Thanked 61 Times in 38 Posts
Gửi tin nhắn qua Yahoo chát tới 353535

[RIGHT][I][B]Nguồn: MathScope.ORG[/B][/I][/RIGHT]
 
353535 is offline  
The Following User Says Thank You to 353535 For This Useful Post:
IMO 2010 (27-11-2010)
Old 31-08-2010, 09:13 PM   #172
353535
Banned
 
Tham gia ngày: Jul 2010
Đến từ: LVT_NB
Bài gởi: 134
Thanks: 3
Thanked 61 Times in 38 Posts
Gửi tin nhắn qua Yahoo chát tới 353535
Đẳng thức xảy ra <=> a=b=c =1 thôi anh
[RIGHT][I][B]Nguồn: MathScope.ORG[/B][/I][/RIGHT]
 
353535 is offline  
The Following User Says Thank You to 353535 For This Useful Post:
IMO 2010 (27-11-2010)
Old 31-08-2010, 09:45 PM   #173
novae
+Thành Viên Danh Dự+
 
novae's Avatar
 
Tham gia ngày: Jul 2010
Đến từ: Event horizon
Bài gởi: 2,453
Thanks: 53
Thanked 3,057 Times in 1,288 Posts
Trường hợp còn lại phải là 2 số bằng 1, 1 số bằng 0
[RIGHT][I][B]Nguồn: MathScope.ORG[/B][/I][/RIGHT]
 
__________________
M.
novae is offline  
The Following User Says Thank You to novae For This Useful Post:
IMO 2010 (27-11-2010)
Old 31-08-2010, 09:49 PM   #174
crystal_liu
+Thành Viên+
 
crystal_liu's Avatar
 
Tham gia ngày: Aug 2010
Đến từ: Akaban
Bài gởi: 353
Thanks: 94
Thanked 199 Times in 141 Posts
Trích:
Nguyên văn bởi novae View Post
Trường hợp còn lại phải là 2 số bằng 1, 1 số bằng 0
Thanks ,lỗi kĩ thuật.Hình như bài này có cách dùng đạo hàm dần cho từng biến song mình không hiểu lắm
PS Đây là BT trên lớp hôm nay .Mình thử dồn b song không ăn thua
[RIGHT][I][B]Nguồn: MathScope.ORG[/B][/I][/RIGHT]
 
__________________
BEAST
crystal_liu is offline  
The Following User Says Thank You to crystal_liu For This Useful Post:
IMO 2010 (27-11-2010)
Old 31-08-2010, 10:01 PM   #175
shido_soichua
Maths is my life
 
shido_soichua's Avatar
 
Tham gia ngày: Oct 2009
Đến từ: Ninh Bình
Bài gởi: 300
Thanks: 31
Thanked 132 Times in 76 Posts
Gửi tin nhắn qua Yahoo chát tới shido_soichua
Trích:
Nguyên văn bởi crystal_liu View Post
Thanks ,lỗi kĩ thuật.Hình như bài này có cách dùng đạo hàm dần cho từng biến song mình không hiểu lắm
PS Đây là BT trên lớp hôm nay .Mình thử dồn b song không ăn thua
Đánh giá chưa đầy đủ đẳng thức $a=a^2=a^3 $ có thể xảy ra với cả $a=0 $ và $a=1 $
[RIGHT][I][B]Nguồn: MathScope.ORG[/B][/I][/RIGHT]
 
__________________
http://luongvantuy.org/forum.php
Chuyên Văn - Lương Văn Tụy
shido_soichua is offline  
The Following User Says Thank You to shido_soichua For This Useful Post:
IMO 2010 (27-11-2010)
Old 31-08-2010, 10:39 PM   #176
hoangduyenkhtn
+Thành Viên+
 
Tham gia ngày: Dec 2008
Bài gởi: 71
Thanks: 56
Thanked 57 Times in 36 Posts
Đẳng thức xảy ra <=> a=b=c =1 thôi anh
bạn thử thay vào bất đẳng thức ngay bước đầu tiên của bạn xem.Mình làm ra đúng dấu bằng giống bạn.bước đầu tiên bạn áp dụng holder đúng ko?
[RIGHT][I][B]Nguồn: MathScope.ORG[/B][/I][/RIGHT]
 
hoangduyenkhtn is offline  
The Following User Says Thank You to hoangduyenkhtn For This Useful Post:
IMO 2010 (27-11-2010)
Old 31-08-2010, 10:47 PM   #177
crystal_liu
+Thành Viên+
 
crystal_liu's Avatar
 
Tham gia ngày: Aug 2010
Đến từ: Akaban
Bài gởi: 353
Thanks: 94
Thanked 199 Times in 141 Posts
Trích:
Nguyên văn bởi hoangduyenkhtn View Post
Đẳng thức xảy ra <=> a=b=c =1 thôi anh
bạn thử thay vào bất đẳng thức ngay bước đầu tiên của bạn xem.Mình làm ra đúng dấu bằng giống bạn.bước đầu tiên bạn áp dụng holder đúng ko?
Thử như Novae 2bieens =1,1bieens =0 là đúng .Bài này đã xong nên chấm dứt ở đây .ai có bài nào ay đưa lên mình chúng mình giao lưu
[RIGHT][I][B]Nguồn: MathScope.ORG[/B][/I][/RIGHT]
 
__________________
BEAST
crystal_liu is offline  
The Following User Says Thank You to crystal_liu For This Useful Post:
IMO 2010 (27-11-2010)
Old 31-08-2010, 11:13 PM   #178
boheoga9999
+Thành Viên+
 
boheoga9999's Avatar
 
Tham gia ngày: Aug 2010
Đến từ: Tp_HCM
Bài gởi: 170
Thanks: 109
Thanked 60 Times in 32 Posts
Th Miniheart4 Một bài bất đẳng thức thú vị

Các bạn thử giải bài này bằng nhiều cách nha:
CMR nêú x , y , z là các số dương thì
$\frac {x^2}{y+z} $+$\frac {y^2}{x+z} $+$\frac {z^2}{x+y} $$\geq $ $\frac {x+y+z}{2} $
[RIGHT][I][B]Nguồn: MathScope.ORG[/B][/I][/RIGHT]
 
__________________
NOTHING IS IMPOSSIBLE
boheoga9999 is offline  
The Following User Says Thank You to boheoga9999 For This Useful Post:
IMO 2010 (27-11-2010)
Old 31-08-2010, 11:22 PM   #179
novae
+Thành Viên Danh Dự+
 
novae's Avatar
 
Tham gia ngày: Jul 2010
Đến từ: Event horizon
Bài gởi: 2,453
Thanks: 53
Thanked 3,057 Times in 1,288 Posts
Cách 1: (Cauchy)
$\frac{x^2}{y+z}+\frac{y+z}{4}\ge x $
$\frac{y^2}{z+x}+\frac{z+x}{4}\ge y $
$\frac{z^2}{x+y}+\frac{x+y}{4}\ge z $
$\Rightarrow VT+\frac{x+y+z}{2}\ge x+y+z \Rightarrow $ đpcm
Cách 2: (Bunyakovski)
$VT\ge \frac{(x+y+z)^2}{2(x+y+z)}=VP $ (đpcm)
[RIGHT][I][B]Nguồn: MathScope.ORG[/B][/I][/RIGHT]
 
__________________
M.
novae is offline  
The Following User Says Thank You to novae For This Useful Post:
IMO 2010 (27-11-2010)
Old 31-08-2010, 11:39 PM   #180
lexuanthang
+Thành Viên+
 
lexuanthang's Avatar
 
Tham gia ngày: Aug 2010
Đến từ: ĐHBKHN
Bài gởi: 26
Thanks: 307
Thanked 20 Times in 15 Posts
Icon7

Trích:
Nguyên văn bởi havgod View Post
mình nghĩ bài này S.O.S đc
biến đổi thành
$\sum ({\frac {a^2} {b} +b-2a})\ge 2(\sqrt {3(a^2+b^2+c^2)}-(a+b+c)}) $
$\sum {\frac {(a-b)^2} {b}\ge (\sum \frac {2(a-b)^2}{\sqrt {3(a^2+b^2+c^2)}+a+b+c}) $
$\sum {(a-b)^2}(\frac 1 b -\frac {2}{\sqrt {3(a^2+b^2+c^2)}+a+b+c})\ge 0 $
dễ thấy $\sqrt{3(a^2+b^2+c^2)}+a+b+c\ge 2b $, từ đó suy ra đpcm
------------------------------

Bài này tương tự bài trên nè
Mình cảm ơn bạn rất nhiều !!!!
[RIGHT][I][B]Nguồn: MathScope.ORG[/B][/I][/RIGHT]
 
lexuanthang is offline  
The Following User Says Thank You to lexuanthang For This Useful Post:
IMO 2010 (27-11-2010)
Ðề tài đã khoá Gởi Ðề Tài Mới

Bookmarks

Tags
bất đẳng thức


Quuyền Hạn Của Bạn
You may not post new threads
You may not post replies
You may not post attachments
You may not edit your posts

BB code is Mở
Smilies đang Mở
[IMG] đang Mở
HTML đang Tắt

Chuyển đến


Múi giờ GMT. Hiện tại là 02:54 AM.


Powered by: vBulletin Copyright ©2000-2024, Jelsoft Enterprises Ltd.
Inactive Reminders By mathscope.org
[page compression: 105.34 k/122.09 k (13.72%)]